Download as pdf or txt
Download as pdf or txt
You are on page 1of 14

Sử dụng AM – GM để chứng minh bất đẳng thức

(Giải bài tập phần chuyên đề)


Đỗ Xuân Trọng

Ngày 10 tháng 1 năm 2023


Về tài liệu này

Cuốn sách Sử dụng AM – GM để chứng minh bất đẳng thức của các tác gả Võ Quốc Bá Cẩn, Trần Quốc Anh là
một trong những cuốn sách đầu tiên tôi đọc khi mới tiếp cận mảng toán Bất đẳng thức. Đó là cuốn sách mà tôi nghĩ tất cả
các bạn học sinh khi học về bất đẳng thức nên có. Các kĩ năng chứng minh giới thiệu trong sách không chỉ giới hạn ở nguyên
bất đẳng thức AM – GM mà chúng còn là những kĩ năng rất quan trọng trong chứng minh bất đẳng thức nói chung. Sau
này, tôi còn đọc nhiều sách và các tài liệu khác về bất đẳng thức, nhưng cuốn sách về AM – GM nói trên vẫn để lại trong tôi
nhiều ấn tượng.
Tài liệu này là lời giải cho phần bài tập ở hai chuyên đề cuối sách. Các bài 1.9 và 2.4 ở dưới đã có lời giải trong sách, nên
tôi không trích dẫn lại, vì ngại gõ. Các bạn có thể tìm đọc chúng qua sách pdf trên mạng.
Chúc mọi người một ngày tốt lành.




2
Chuyên đề 1
Sắp thứ tự các biến: Đơn giản mà hiệu quả

Bài 1.1. Chứng minh rằng với mọi số thực không âm 𝑎, 𝑏, 𝑐, ta luôn có
3 2 2 2
(𝑎2 + 𝑏2 + 𝑐2 ) ≥ 27 (𝑎 − 𝑏) (𝑏 − 𝑐) (𝑐 − 𝑎) .

Lời giải. Không mất tính tổng quát, giả sử 𝑎 ≥ 𝑏 ≥ 𝑐 ≥ 0. Khi đó, dễ thấy
2 2
(𝑏 − 𝑐) ≤ 𝑏2 , (𝑐 − 𝑎) ≤ 𝑎2 ,

nên suy ra

2 2 2 2 2 3 3 3
27 (𝑎 − 𝑏) (𝑏 − 𝑐) (𝑐 − 𝑎) ≤ 27𝑎2 𝑏2 (𝑎 − 𝑏) ≤ [𝑎𝑏 + 𝑎𝑏 + (𝑎 − 𝑏) ] = (𝑎2 + 𝑏2 ) ≤ (𝑎2 + 𝑏2 + 𝑐2 ) .


Đẳng thức xảy ra khi (𝑎, 𝑏, 𝑐) ∼ ( 3+2 5 , 1, 0). 

Bài 1.2. Chứng minh rằng với mọi số thực 𝑎, 𝑏, 𝑐, ta luôn có


3 2 2 2
(𝑎2 + 𝑏2 + 𝑐2 ) ≥ 2 (𝑎 − 𝑏) (𝑏 − 𝑐) (𝑐 − 𝑎) .

Lời giải. Ta có
2 2 2 2 2 2 2
(𝑎 + 𝑏 + 𝑐) + (𝑎 − 𝑏) + (𝑏 − 𝑐) + (𝑐 − 𝑎) (𝑎 − 𝑏) + (𝑏 − 𝑐) + (𝑐 − 𝑎) 2
𝑎2 +𝑏2 +𝑐2 = ≥ = (𝑎2 + 𝑏2 + 𝑐2 − 𝑎𝑏 − 𝑏𝑐 − 𝑐𝑎) ≥ 0,
3 3 3
nên bất đẳng thức đã cho được chứng minh, nếu ta chứng minh được

8 2 3 2 2 2
(𝑎 + 𝑏2 + 𝑐2 − 𝑎𝑏 − 𝑏𝑐 − 𝑐𝑎) ≥ 2 (𝑎 − 𝑏) (𝑏 − 𝑐) (𝑐 − 𝑎) .
27
Nhưng bất đẳng thức này đúng, vì nó tương đương với
2 2 2
(𝑎 + 𝑏 − 2𝑐) (𝑏 + 𝑐 − 2𝑎) (𝑐 + 𝑎 − 2𝑏) ≥ 0.

Đẳng thức xảy ra khi (𝑎, 𝑏, 𝑐) ∼ (1, −1, 0). 

3
CHƯƠNG 1. SẮP THỨ TỰ CÁC BIẾN: ĐƠN GIẢN MÀ HIỆU QUẢ

Bài 1.3. Cho 𝑎, 𝑏, 𝑐 là các số thực không âm có tổng bằng 2. Chứng minh rằng

𝑎3 (𝑏2 + 𝑐2 ) + 𝑏3 (𝑐2 + 𝑎2 ) + 𝑐3 (𝑎2 + 𝑏2 ) ≤ 2.

Lời giải. Rõ ràng 𝑎 + 𝑏 ≤ 2, 𝑏 + 𝑐 ≤ 2 và 𝑐 + 𝑎 ≤ 2. Suy ra

𝑎2 + 𝑏2 ≥ 2𝑎𝑏 ≥ 𝑎𝑏 (𝑎 + 𝑏) ,

𝑎𝑏 (𝑎2 + 𝑏2 ) ≥ 𝑎2 𝑏2 (𝑎 + 𝑏) .
Tương tự,
𝑏𝑐 (𝑏2 + 𝑐2 ) ≥ 𝑏2 𝑐2 (𝑏 + 𝑐) , 𝑐𝑎 (𝑐2 + 𝑎2 ) ≥ 𝑐2 𝑎2 (𝑐 + 𝑎) .
Các bất đẳng thức trên dẫn tới
2
3 2 2 2 2 2 (∑ 𝑎2 + 2 ∑ 𝑎𝑏)
2 (𝑎 + 𝑏 + 𝑐)
4
2
∑ 𝑎 (𝑏 + 𝑐 ) = ∑ 𝑎 𝑏 (𝑎 + 𝑏) ≤ ∑ 𝑎𝑏 (𝑎 + 𝑏 ) ≤ (∑ 𝑎𝑏) (∑ 𝑎 ) ≤ = = 2.
8 8
Đẳng thức xảy ra khi (𝑎, 𝑏, 𝑐) ∼ (1, 1, 0). 

Bài 1.4. Cho các số thực 𝑎, 𝑏, 𝑐 đôi một khác nhau, thỏa mãn 𝑎 + 𝑏 + 𝑐 = 0. Chứng minh rằng

2 1 1 1 33
(𝑎𝑏 + 𝑏𝑐 + 𝑐𝑎) [ + + ]≥ .
(𝑎 − 𝑏)
4
(𝑏 − 𝑐)
4
(𝑐 − 𝑎)
4 16

Lời giải. Theo nguyên lý Dirichlet, trong ba số 𝑎, 𝑏, 𝑐, tồn tại hai số cùng dấu. Ta có thể giả sử hai số đó là 𝑎 và 𝑏 để thu được
𝑎𝑏 ≥ 0. Khi đó, bằng cách thay 𝑐 = −𝑎 − 𝑏, bất đẳng thức trên chỉ là một bất đẳng thức thuần nhất với hai biến 𝑎, 𝑏 và sau khi
quy đồng mẫu số chung, ta thu được bất đẳng thức hiển nhiên
2 3
𝑎2 𝑏2 (𝑎 + 𝑏) [152 (𝑎6 + 𝑏6 ) + 456𝑎𝑏 (𝑎4 + 𝑏4 ) + 470 (𝑎𝑏) + 615𝑎2 𝑏2 (𝑎2 + 𝑏2 )] ≥ 0.

Đẳng thức xảy ra khi (𝑎, 𝑏, 𝑐) ∼ (1, −1, 0). 

Bài 1.5. Tìm hằng số 𝑘 tốt nhất sao cho bất đẳng thức sau đúng với mọi số thực không âm 𝑎, 𝑏, 𝑐.
5
(𝑎 + 𝑏 + 𝑐) ≥ 𝑘 (𝑎2 + 𝑏2 + 𝑐2 ) (𝑎 − 𝑏) (𝑏 − 𝑐) (𝑐 − 𝑎) .

Lời giải. Cho


3 + √5 + √10 + 6√5
(𝑎, 𝑏, 𝑐) = (0, 1, ),
2

ta thu được 𝑘 ≤ 10√4 5. Do đó, ta sẽ chứng minh giá trị lớn nhất của 𝑘 là 10√4 5, bằng cách chứng minh
5
(𝑎 + 𝑏 + 𝑐) ≥ 10√5 (𝑎2 + 𝑏2 + 𝑐2 ) (𝑎 − 𝑏) (𝑏 − 𝑐) (𝑐 − 𝑎) . (1.1)
4

Rõ ràng, ta chỉ cần xét trường hợp 𝑎 ≤ 𝑏 ≤ 𝑐. Bình phương hai vế của (1.1), ta có
10 2 2 2 2
(𝑎 + 𝑏 + 𝑐) ≥ 100√5 (𝑎2 + 𝑏2 + 𝑐2 ) (𝑎 − 𝑏) (𝑏 − 𝑐) (𝑐 − 𝑎) .

Đỗ Xuân Trọng 4
CHƯƠNG 1. SẮP THỨ TỰ CÁC BIẾN: ĐƠN GIẢN MÀ HIỆU QUẢ

3+√5+√10+6√5
Đặt 𝑡 = 2 , là nghiệm của phương trình

𝑡4 − 6𝑡3 + 6𝑡2 − 6𝑡 + 1 = 0.

Theo bất đẳng thức AM – GM,

2 2
2 2 2 2 100√5𝑡2 (𝑡 − 1) 2
2 𝑡 +1 𝑡2 + 1 2
100√5 (𝑎2 + 𝑏2 + 𝑐2 ) (𝑎 − 𝑏) (𝑏 − 𝑐) (𝑐 − 𝑎) = (𝑎2 + 𝑏2 + 𝑐2 ) [ (𝑏 − 𝑎) (𝑐 − 𝑎)] [ (𝑏 − 𝑐) ]
2
(𝑡 + 1)
3 𝑡 (𝑡 − 1)
2

5
100√5𝑡2 (𝑡 − 1)
2
2 2 (𝑡2 + 1)
2 2 𝑡2 + 1 2
≤ [2 (𝑎 + 𝑏 + 𝑐 ) + (𝑏 − 𝑎) (𝑐 − 𝑎) + (𝑏 − 𝑐) ]
55 (𝑡2 + 1)
3 𝑡 (𝑡 − 1)
2

5
100√5𝑡2 (𝑡 − 1)
2
2 2 (𝑡2 + 1)
2 2 𝑡2 + 1 2
≤ [2 (𝑎 + 𝑏 + 𝑐 ) + 𝑏𝑐 + (𝑏 − 𝑐) ]
55 (𝑡2 + 1)
3 𝑡 (𝑡 − 1)
2

2 5
100√5𝑡2 (𝑡 − 1) 𝑡2 + 1 𝑡2 + 1 𝑡2 + 1
= [2𝑎2 + (2 + ) (𝑏2 + 𝑐2 ) + ( − ) 2𝑏𝑐] .
55 (𝑡2 + 1)
3
(𝑡 − 1)
2 𝑡 (𝑡 − 1)
2

Bởi vì 𝑡 − 6𝑡 + 6𝑡 − 6𝑡 + 1 = 0, ta có
4 3 2

𝑡2 + 1 𝑡2 + 1 𝑡2 + 1
2+ = − .
(𝑡 − 1)
2 𝑡 (𝑡 − 1)
2

Mặt khác, dễ thấy rằng

𝑡2 + 1 2 𝑡2 + 1 2 𝑡2 + 1 𝑡2 + 1 2
2𝑎2 + (2 + ) (𝑏 + 𝑐) ≤ (2 + ) (𝑎 + 𝑏 + 𝑐) = ( − ) (𝑎 + 𝑏 + 𝑐) ,
(𝑡 − 1)
2
(𝑡 − 1)
2 𝑡 (𝑡 − 1)
2

nên suy ra

2 5
2 2 2 2 100√5𝑡2 (𝑡 − 1) 𝑡2 + 1 𝑡2 + 1 2
100√5 (𝑎2 + 𝑏2 + 𝑐2 ) (𝑎 − 𝑏) (𝑏 − 𝑐) (𝑐 − 𝑎) ≤ [( − ) (𝑎 + 𝑏 + 𝑐) ]
5 2
5 (𝑡 + 1)
3 𝑡 (𝑡 − 1)
2

2 5
100√5 (𝑡2 + 1) (𝑡2 − 3𝑡 + 1) 10
= 8
(𝑎 + 𝑏 + 𝑐)
55 𝑡3 (𝑡 − 1)
Bây giờ, để ý rằng
4
𝑡2 − 3𝑡 + 1 = √5 ⋅ 𝑡, (𝑡 − 1) = 2𝑡 (𝑡2 + 1) ,

ta có
2 5 2 5
100√5 (𝑡2 + 1) (𝑡2 − 3𝑡 + 1) 100√5 (𝑡2 + 1) (√5 ⋅ 𝑡)
8
= 2
= 1,
55 𝑡3 (𝑡 − 1) 55 𝑡3 (2𝑡 (𝑡2 + 1))
và do đó,
2 2 2 2 10
100√5 (𝑎2 + 𝑏2 + 𝑐2 ) (𝑎 − 𝑏) (𝑏 − 𝑐) (𝑐 − 𝑎) ≤ (𝑎 + 𝑏 + 𝑐) .

Vậy ta kết luận giá trị lớn nhất của 𝑘 là 10√4 5. 

Bài 1.6. Chứng minh rằng với mọi số thực không âm 𝑎, 𝑏, 𝑐, ta luôn có
3
(𝑎2 + 𝑎𝑏 + 𝑏𝑐) (𝑏2 + 𝑏𝑐 + 𝑐𝑎) (𝑐2 + 𝑐𝑎 + 𝑎𝑏) ≥ (𝑎𝑏 + 𝑏𝑐 + 𝑐𝑎) .

Đỗ Xuân Trọng 5
CHƯƠNG 1. SẮP THỨ TỰ CÁC BIẾN: ĐƠN GIẢN MÀ HIỆU QUẢ

Lời giải. Không mất tính tổng quát, giả sử 𝑏 = min {𝑎, 𝑏, 𝑐}. Theo bất đẳng thức Cauchy – Schwarz,
2
(𝑎2 + 𝑎𝑏 + 𝑏𝑐) (𝑐2 + 𝑎𝑏 + 𝑏𝑐) ≥ (𝑎𝑐 + 𝑎𝑏 + 𝑏𝑐) .

Do đó, bài toán sẽ được chứng minh nếu ta chứng minh được

(𝑏2 + 𝑏𝑐 + 𝑐𝑎) (𝑐2 + 𝑐𝑎 + 𝑎𝑏) ≥ (𝑐2 + 𝑎𝑏 + 𝑏𝑐) (𝑎𝑏 + 𝑏𝑐 + 𝑐𝑎) ,

𝑎 (𝑏 + 𝑐) (𝑎 − 𝑏) (𝑐 − 𝑏) ≥ 0,
hiển nhiên đúng với giả thiết 𝑏 = min {𝑎, 𝑏, 𝑐}. 

Bài 1.7. Cho các số thực không âm 𝑎, 𝑏, 𝑐, thỏa mãn 𝑎𝑏 + 𝑏𝑐 + 𝑐𝑎 > 0. Chứng minh

2 (𝑎2 + 𝑏2 + 𝑐2 ) 𝑎𝑏𝑐
2
+ ≥ 1.
(𝑎 + 𝑏 + 𝑐) 𝑎2 𝑏 + 𝑏2 𝑐 + 𝑐2 𝑎

Lời giải. Không mất tính tổng quát, giả sử 𝑏 là số nằm giữa 𝑎 và 𝑐. Khi đó (𝑏 − 𝑎) (𝑏 − 𝑐) ≤ 0, nên

𝑎2 𝑏 + 𝑏2 𝑐 + 𝑐2 𝑎 = 𝑏 (𝑐2 + 𝑐𝑎 + 𝑎2 ) + 𝑐 (𝑏 − 𝑎) (𝑏 − 𝑐) ≤ 𝑏 (𝑐2 + 𝑐𝑎 + 𝑎2 ) ,

dẫn tới
𝑎𝑏𝑐 𝑎𝑐
≥ .
𝑎2 𝑏 + 𝑏2 𝑐 + 𝑐2 𝑎 𝑎2 + 𝑎𝑐 + 𝑐2
Do đó, bài toán được quy về chứng minh

2 (𝑎2 + 𝑏2 + 𝑐2 ) 𝑎𝑐
2
+ ≥ 1.
(𝑎 + 𝑏 + 𝑐) 𝑎2 + 𝑎𝑐 + 𝑐2

Bất đẳng thức này đúng, vì nó tương đương với


2
(𝑎2 − 𝑎𝑏 − 𝑏𝑐 + 𝑐2 )
2
≥ 0.
(𝑎2 + 𝑎𝑐 + 𝑐2 ) (𝑎 + 𝑏 + 𝑐)

Đẳng thức xảy ra khi 𝑎 = 𝑏 = 𝑐, hoặc (𝑎, 𝑏, 𝑐) ∼ (1, 1, 0). 

Bài 1.8. Cho các số không âm 𝑎, 𝑏, 𝑐, thỏa mãn 𝑎 + 2𝑏 + 3𝑐 = 4. Chứng minh

(2𝑎2 + 𝑏𝑐) (2𝑏2 + 𝑐𝑎) (2𝑐2 + 𝑎𝑏) ≤ 32.

Lời giải. Đẳng thức xảy ra khi (𝑎, 𝑏, 𝑐) = (2, 1, 0). Sử dụng bất đẳng thức AM – GM, ta có
1
(2𝑎2 + 𝑏𝑐) (2𝑏2 + 𝑐𝑎) (2𝑐2 + 𝑎𝑏) = √(2𝑎2 + 𝑏𝑐) (2𝑏2 + 𝑐𝑎) ⋅ √(2𝑎2 + 𝑏𝑐) (2𝑏2 + 𝑐𝑎) ⋅ 2 (2𝑐2 + 𝑎𝑏)
2
1 3
≤ [2√(2𝑎2 + 𝑏𝑐) (2𝑏2 + 𝑐𝑎) + 2 (2𝑐2 + 𝑎𝑏)]
54
4 3
= [√(2𝑎2 + 𝑏𝑐) (2𝑏2 + 𝑐𝑎) + 2𝑐2 + 𝑎𝑏] .
27
Vậy, ta chỉ cần chứng minh
√(2𝑎2 + 𝑏𝑐) (2𝑏2 + 𝑐𝑎) + 2𝑐2 + 𝑎𝑏 ≤ 6,

Đỗ Xuân Trọng 6
CHƯƠNG 1. SẮP THỨ TỰ CÁC BIẾN: ĐƠN GIẢN MÀ HIỆU QUẢ

2
8 [√(2𝑎2 + 𝑏𝑐) (2𝑏2 + 𝑐𝑎) + 2𝑐2 + 𝑎𝑏] ≤ 3 (𝑎 + 2𝑏 + 3𝑐) ,

8√(2𝑎2 + 𝑏𝑐) (2𝑏2 + 𝑐𝑎) ≤ 11𝑐2 + 18𝑐 (𝑎 + 2𝑏) + 3𝑎2 + 12𝑏2 + 4𝑎𝑏,
2
64 (2𝑎2 + 𝑏𝑐) (2𝑏2 + 𝑐𝑎) ≤ [11𝑐2 + 18𝑐 (𝑎 + 2𝑏) + 3𝑎2 + 12𝑏2 + 4𝑎𝑏] .

Để ý là
2 2
𝑉𝛲 ≥ 182 (𝑎 + 2𝑏) 𝑐2 + 36𝑐 (𝑎 + 2𝑏) (3𝑎2 + 12𝑏2 + 4𝑎𝑏) + 2 ⋅ 11𝑐2 ⋅ 4𝑎𝑏 + (3𝑎2 + 4𝑎𝑏 + 12𝑏2 ) ,
2 2
≥ 182 (𝑎 + 𝑏) 𝑐2 + 36𝑐 (𝑎 + 𝑏) (3𝑎2 + 12𝑏2 + 4𝑎𝑏) + 64𝑎𝑏𝑐2 + (3𝑎2 + 4𝑎𝑏 + 12𝑏2 ) ,

ta chỉ cần chứng minh được


2 2
64 (2𝑎2 + 𝑏𝑐) (2𝑏2 + 𝑐𝑎) ≤ 182 (𝑎 + 𝑏) 𝑐2 + 36𝑐 (𝑎 + 𝑏) (3𝑎2 + 12𝑏2 + 4𝑎𝑏) + 64𝑎𝑏𝑐2 + (3𝑎2 + 4𝑎𝑏 + 12𝑏2 ) ,
2 2
182 (𝑎 + 𝑏) 𝑐2 + 36𝑐 (𝑎 + 𝑏) (3𝑎2 + 12𝑏2 + 4𝑎𝑏) + 3 (𝑎 − 2𝑏) (3𝑎2 + 12𝑏2 + 20𝑎𝑏) ≥ 128𝑐 (𝑎3 + 𝑏3 ) .

Lại theo bất đẳng thức 𝛢𝛭 − 𝐺𝛭,


2 2 2 2 2
182 (𝑎 + 𝑏) 𝑐2 + 3 (𝑎 − 2𝑏) (3𝑎2 + 12𝑏2 + 20𝑎𝑏) ≥ 182 (𝑎 + 𝑏) 𝑐2 + 3 (𝑎 − 2𝑏) ⋅ 3 (𝑎 + 𝑏)
2
≥ 108𝑐 (𝑎 + 𝑏) |𝑎 − 2𝑏| ,

nên ta chỉ cần chứng minh


2
108𝑐 (𝑎 + 𝑏) |𝑎 − 2𝑏| + 36𝑐 (𝑎 + 𝑏) (3𝑎2 + 12𝑏2 + 4𝑎𝑏) ≥ 128𝑐 (𝑎3 + 𝑏3 ) ,

27 (𝑎 + 𝑏) |𝑎 − 2𝑏| + 9 (3𝑎2 + 12𝑏2 + 4𝑎𝑏) ≥ 32 (𝑎2 − 𝑎𝑏 + 𝑏2 ) .

• Nếu 𝑎 ≥ 2𝑏 thì

𝑉𝛲 = 27 (𝑎 + 𝑏) (𝑎 − 2𝑏) + 9 (3𝑎2 + 12𝑏2 + 4𝑎𝑏) = 54𝑎2 + 54𝑏2 + 9𝑎𝑏 > 32 (𝑎2 − 𝑎𝑏 + 𝑏2 ) .

• Nếu 𝑎 < 2𝑏 thì

𝑉𝛲 = 27 (𝑎 + 𝑏) (2𝑏 − 𝑎)+9 (3𝑎2 + 12𝑏2 + 4𝑎𝑏) = 32 (𝑎2 − 𝑎𝑏 + 𝑏2 )+2𝑏2 +95𝑎𝑏+32 (4𝑏2 − 𝑎2 ) > 32 (𝑎2 − 𝑎𝑏 + 𝑏2 ) .

Bài toán được chứng minh xong. 

Bài 1.9. Cho 𝑎, 𝑏, 𝑐 là các số thực không âm, thỏa mãn 𝑎 + 𝑏 + 𝑐 = 3 và 𝑎𝑏 + 𝑏𝑐 + 𝑐𝑎 > 0. Chứng minh rằng

𝑎𝑏 𝑏𝑐 𝑐𝑎 1 (𝑎 + 𝑏) (𝑏 + 𝑐) (𝑐 + 𝑎)
+ + + ≥ .
√𝑏 + 𝑐 √𝑐 + 𝑎 √𝑎 + 𝑏 √2 2√ 2

Lời giải. Đây là bài 68 của chương 3 (xem trang 162 của sách). 

Bài 1.10. Cho các số thực dương 𝑎, 𝑏, 𝑐, 𝑑. Chứng minh rằng


2
(𝑎 + 𝑏) (𝑎 + 𝑏 + 𝑐) (𝑎 + 𝑏 + 𝑐 + 𝑑)
≥ 64.
𝑎𝑏𝑐𝑑

Đỗ Xuân Trọng 7
CHƯƠNG 1. SẮP THỨ TỰ CÁC BIẾN: ĐƠN GIẢN MÀ HIỆU QUẢ

2
Lời giải. Sử dụng liên tiếp bất đẳng thức (𝑥 + 𝑦) ≥ 4𝑥𝑦, ta có
2 2
(𝑎 + 𝑏) (𝑎 + 𝑏 + 𝑐) (𝑎 + 𝑏 + 𝑐 + 𝑑) ≥ (𝑎 + 𝑏) (𝑎 + 𝑏 + 𝑐) ⋅ 4𝑑 (𝑎 + 𝑏 + 𝑐) = 4𝑑 (𝑎 + 𝑏) (𝑎 + 𝑏 + 𝑐)
2
≥ 4𝑑 (𝑎 + 𝑏) ⋅ 4𝑐 (𝑎 + 𝑏) = 16𝑑𝑐 (𝑎 + 𝑏) ≥ 16𝑐𝑑 ⋅ 4𝑎𝑏 = 64𝑎𝑏𝑐𝑑,

đó chính là điều phải chứng minh. 

Bài 1.11. Cho các số thực không âm 𝑎, 𝑏, 𝑐, thỏa mãn 𝑎 + 𝑏 + 𝑐 = 1. Chứng minh
108
(𝑎2 + 𝑎𝑏 + 𝑏𝑐) (𝑏2 + 𝑏𝑐 + 𝑐𝑎) (𝑐2 + 𝑐𝑎 + 𝑎𝑏) − 𝑎𝑏𝑐 ≤ .
3125

Lời giải. Không mất tính tổng quát, giả sử 𝑐 = min {𝑎, 𝑏, 𝑐}. Ta sẽ chứng minh rằng vế trái lớn nhất khi 𝑏 ≥ 𝑎 ≥ 𝑐. Thật vậy,
gọi 𝛲 (𝑎, 𝑏, 𝑐) là vế trái. Nếu 𝑎 ≥ 𝑏 ≥ 𝑐, ta có

𝛲 (𝑏, 𝑎, 𝑐) − 𝛲 (𝑎, 𝑏, 𝑐) = (𝑎 − 𝑏) (𝑏 − 𝑐) (𝑎 − 𝑐) (𝑎𝑏𝑐 + ∑ 𝑎𝑏 (𝑎 + 𝑏)) ≥ 0,

nên trong biểu thức 𝛲 (𝑎, 𝑏, 𝑐) ban đầu, bằng cách đổi chỗ 𝑎 và 𝑏 cho nhau, ta sẽ thu được biểu thức mới 𝛲 (𝑏, 𝑎, 𝑐) có giá trị
không nhỏ hơn nó. Bây giờ, với 𝑏 ≥ 𝑎 ≥ 𝑐, ta có

(𝑎2 + 𝑎𝑏 + 𝑏𝑐) (𝑐2 + 𝑐𝑎 + 𝑎𝑏) − 𝑎 (𝑎 + 𝑏 + 𝑐) (𝑐2 + 𝑎𝑏 + 𝑏𝑐) = 𝑐 (𝑎 + 𝑐) (𝑎 − 𝑏) (𝑎 − 𝑐) ≤ 0,

nên ta chỉ cần chứng minh được

108
𝑎 (𝑎 + 𝑏 + 𝑐) (𝑐2 + 𝑎𝑏 + 𝑏𝑐) (𝑏2 + 𝑏𝑐 + 𝑐𝑎) − 𝑎𝑏𝑐 ≤ . (1.2)
3125
Với giả thiết 𝑎 + 𝑏 + 𝑐 = 1, bất đẳng thức (1.2) tương đương với

108
𝑎 (𝑐2 + 𝑏 − 𝑏2 ) (𝑏2 + 𝑐 − 𝑐2 ) − 𝑎𝑏𝑐 ≤ ,
3125
2 108
𝑎 (𝑏 − 𝑐) (𝑏 + 𝑐) (1 − 𝑏 − 𝑐) ≤ ,
3125
2 108
𝑎2 (𝑏 − 𝑐) (𝑏 + 𝑐) ≤ .
3125
Sử dụng bất đẳng thức AM – GM, ta có
5
2 1 2 2 [(3𝑎) ⋅ 2 + 2 (𝑏 − 𝑐) ⋅ 2 + 2 (𝑏 + 𝑐)]
𝑎2 (𝑏 − 𝑐) (𝑏 + 𝑐) = (3𝑎) ⋅ [2 (𝑏 − 𝑐)] ⋅ 2 (𝑏 + 𝑐) ≤
72 72 ⋅ 55
5 5 5
(6𝑎 + 6𝑏 − 2𝑐) (6𝑎 + 6𝑏 + 6𝑐) 6 108
= ≤ = = .
72 ⋅ 5 5 72 ⋅ 5 5 72 ⋅ 5 5 3125
2 3
Đẳng thức xảy ra khi (𝑎, 𝑏, 𝑐) = ( , , 0) và các hoán vị tương ứng. 
5 5

Bài 1.12. Cho 𝑥, 𝑦, 𝑧 là các số không âm, thỏa mãn 𝑥 + 𝑦 + 𝑧 = 1. Chứng minh
2
2 (𝑥2 + 𝑦2 + 𝑧2 + 1) (𝑥𝑦3 + 𝑦𝑧3 + 𝑧𝑥3 + 𝑥𝑦𝑧) ≤ (𝑥2 + 𝑦2 + 𝑧2 + 3𝑥𝑦𝑧) .

Đỗ Xuân Trọng 8
CHƯƠNG 1. SẮP THỨ TỰ CÁC BIẾN: ĐƠN GIẢN MÀ HIỆU QUẢ

Lời giải. Viết lại bất đẳng thức ở dạng thuần nhất như sau.
2 2
2 [𝑥2 + 𝑦2 + 𝑧2 + (𝑥 + 𝑦 + 𝑧) ] [𝑥𝑦3 + 𝑦𝑧3 + 𝑧𝑥3 + 𝑥𝑦𝑧 (𝑥 + 𝑦 + 𝑧)] ≤ [(𝑥2 + 𝑦2 + 𝑧2 ) (𝑥 + 𝑦 + 𝑧) + 3𝑥𝑦𝑧] ,

2
4 (𝑥2 + 𝑦2 + 𝑧2 + 𝑥𝑦 + 𝑦𝑧 + 𝑧𝑥) [𝑥𝑦3 + 𝑦𝑧3 + 𝑧𝑥3 + 𝑥𝑦𝑧 (𝑥 + 𝑦 + 𝑧)] ≤ [(𝑥2 + 𝑦2 + 𝑧2 ) (𝑥 + 𝑦 + 𝑧) + 3𝑥𝑦𝑧] ,

Không mất tính tổng quát, giả sử 𝑦 là số nằm giữa 𝑥 và 𝑧, khi đó (𝑦 − 𝑥) (𝑦 − 𝑧) ≤ 0. Tiếp theo, áp dụng bất đẳng thức
2
4𝛸𝑌 ≤ (𝛸 + 𝑌) ,
2
𝑥𝑦3 + 𝑦𝑧3 + 𝑧𝑥3 + 𝑥𝑦𝑧 (𝑥 + 𝑦 + 𝑧)
𝑉𝛵 ≤ [𝑦 (𝑥2 + 𝑦2 + 𝑧2 + 𝑥𝑦 + 𝑦𝑧 + 𝑧𝑥) + ] ,
𝑦

ta quy bài toán về chứng minh

𝑥𝑦3 + 𝑦𝑧3 + 𝑧𝑥3 + 𝑥𝑦𝑧 (𝑥 + 𝑦 + 𝑧)


𝑦 (𝑥2 + 𝑦2 + 𝑧2 + 𝑥𝑦 + 𝑦𝑧 + 𝑧𝑥) + ≤ (𝑥2 + 𝑦2 + 𝑧2 ) (𝑥 + 𝑦 + 𝑧) + 3𝑥𝑦𝑧.
𝑦

Bất đẳng thức này được thu gọn thành


𝑥 (𝑥 + 𝑦) (𝑦 − 𝑥) (𝑦 − 𝑧)
≤ 0,
𝑦
hiển nhiên đúng. 

Bài 1.13. Cho 𝑎, 𝑏, 𝑐 là các số thực không âm, thỏa mãn 𝑎 + 𝑏 + 𝑐 = 1. Tìm giá trị lớn nhất của biểu thức

𝛲 = 𝑎2 𝑏3 + 𝑏2 𝑐3 + 𝑐2 𝑎3 .

Lời giải. Không mất tính tổng quát, giả sử 𝑐 = min {𝑎, 𝑏, 𝑐}. Tương tự như bài 1.11, ta chỉ cần xét trường hợp 𝑏 ≥ 𝑎 ≥ 𝑐.
Khi đó, ta sẽ chứng minh
4𝑐 2 𝑐 3
𝑎2 𝑏3 + 𝑏2 𝑐3 + 𝑐2 𝑎3 ≤ (𝑎 + ) (𝑏 + ) .
5 5
Thật vậy,
4𝑐 2 𝑐 3 𝑐
(𝑎 + ) (𝑏 + ) − (𝑎2 𝑏3 + 𝑏2 𝑐3 + 𝑐2 𝑎3 ) = ⋅ 𝛢,
5 5 3125
trong đó 𝛢 là một biểu thức không âm,

𝛢 = 2000𝑏
⏟⏟⏟ 3 𝑐⏟ 1875𝑎2 𝑏2 −
+⏟⏟⏟ ⏟3125𝑎
⏟⏟⏟ 3 𝑐 + 5000𝑎𝑏
⏟⏟ ⏟⏟ 3 − 1925𝑏
⏟⏟⏟ 2 𝑐2 +375𝑎2 𝑏𝑐+25𝑎2 𝑐2 +3000𝑎𝑏2 𝑐+600𝑎𝑏𝑐2 +40𝑎𝑐3 +240𝑏𝑐3 +16𝑐4 .
≥0 ≥0

Mặt khác, áp dụng bất đẳng thức AM – GM, ta có


5
4𝑐 2 𝑐 3 1 4𝑐 2 𝑐 3 1 4𝑐 𝑐 5
65 (𝑎 + 𝑏 + 𝑐) 108
(𝑎 + ) (𝑏 + ) = [3 (𝑎 + )] [2 (𝑏 + )] ≤ [3 (𝑎 + ) ⋅ 2 + 2 (𝑏 + ) ⋅ 3] = = ,
5 5 72 5 5 72 ⋅ 5 5 5 5 72 ⋅ 5 5 3125
nên suy ra
4𝑐 2 𝑐 3 108
𝑎2 𝑏3 + 𝑏2 𝑐3 + 𝑐2 𝑎3 ≤ (𝑎 + ) (𝑏 + ) ≤ .
5 5 3125
2 3
Đẳng thức xảy ra khi (𝑎, 𝑏, 𝑐) = ( , , 0), nên ta kết luận giá trị lớn nhất của 𝛲 là 108/3125. 
5 5

Đỗ Xuân Trọng 9
Chuyên đề 2
Những bài toán lí thú xoay quanh một đại lượng hoán vị

Bài 2.1. Chứng minh rằng với mọi số thực dương 𝑎, 𝑏, 𝑐, ta luôn có
2
(𝑎 + 𝑏 + 𝑐) 1 9
2
≥∑ ≥ .
(𝑎𝑏 + 𝑏𝑐 + 𝑐𝑎) 𝑎2 + 𝑎𝑏 + 𝑏𝑐 (𝑎 + 𝑏 + 𝑐)2

Lời giải. Với dấu ≥ thứ hai, áp dụng trực tiếp bất đẳng thức 1/𝑥 + 1/𝑦 + 1/𝑧 ≥ 9/ (𝑥 + 𝑦 + 𝑧),

1 9 9
∑ ≥ = .
𝑎2 + 𝑎𝑏 + 𝑏𝑐 ∑ (𝑎2 + 𝑎𝑏 + 𝑏𝑐) (𝑎 + 𝑏 + 𝑐)2

Với dấu ≥ thứ nhất, áp dụng bất đẳng thức Cauchy – Schwarz,
2
1 𝑐2 + 𝑎𝑏 + 𝑏𝑐 𝑐2 + 𝑎𝑏 + 𝑏𝑐 (𝑎 + 𝑏 + 𝑐)
∑ 2
=∑ 2 2
≤∑ 2
= 2
.
𝑎 + 𝑎𝑏 + 𝑏𝑐 (𝑎 + 𝑎𝑏 + 𝑏𝑐) (𝑐 + 𝑎𝑏 + 𝑏𝑐) (𝑎𝑐 + 𝑎𝑏 + 𝑏𝑐) (𝑎𝑏 + 𝑏𝑐 + 𝑐𝑎)

Ở cả hai bất đẳng thức, đẳng thức đều xảy ra khi 𝑎 = 𝑏 = 𝑐. 

Bài 2.2. Cho các số thực dương 𝑎, 𝑏, 𝑐, thỏa mãn (4𝑎 + 5𝑏) (4𝑏 + 5𝑐) (4𝑐 + 5𝑎) = 729. Chứng minh rằng

𝑎𝑏𝑐 (𝑎2 + 𝑎𝑏 + 𝑏𝑐) (𝑏2 + 𝑏𝑐 + 𝑐𝑎) (𝑐2 + 𝑐𝑎 + 𝑎𝑏) ≤ 27.

Lời giải. Bất đẳng thức đã cho tương đương với

35√3 𝑎𝑏𝑐 (𝑎2 + 𝑎𝑏 + 𝑏𝑐) (𝑏2 + 𝑏𝑐 + 𝑐𝑎) (𝑐2 + 𝑐𝑎 + 𝑎𝑏) ≤ (4𝑎 + 5𝑏) (4𝑏 + 5𝑐) (4𝑐 + 5𝑎) . (2.1)

Theo bất đẳng thức AM – GM,

33𝑎𝑏𝑐
(4𝑎 + 5𝑏) (4𝑏 + 5𝑐) (4𝑐 + 5𝑎) = 99𝑎𝑏𝑐 + ∑ (40𝑏 + 30𝑐) (𝑎2 + 𝑎𝑏 + 𝑏𝑐) = ∑ (40𝑏 + 30𝑐 + ) (𝑎2 + 𝑎𝑏 + 𝑏𝑐)
𝑎2 + 𝑎𝑏 + 𝑏𝑐

33𝑎𝑏𝑐
≥ 3√
3
∏ (𝑎2 + 𝑎𝑏 + 𝑏𝑐) ⋅ ∏ (40𝑏 + 30𝑐 + ).
𝑎2 + 𝑎𝑏 + 𝑏𝑐
Do đó, ta quy (2.1) về chứng minh
33𝑎𝑏𝑐
∏ (40𝑏 + 30𝑐 + ) ≥ 312 𝑎𝑏𝑐.
𝑎2 + 𝑎𝑏 + 𝑏𝑐

10
CHƯƠNG 2. NHỮNG BÀI TOÁN LÍ THÚ XOAY QUANH MỘT ĐẠI LƯỢNG HOÁN VỊ

Sử dụng các bất đẳng thức Holder và AM – GM, ta có


3
33𝑎𝑏𝑐 33𝑎𝑏𝑐
) ≥ [10√𝑎𝑏𝑐 + 30√3 ∏ (𝑏 + 𝑐) +
3
𝑉𝛵 = ∏ (10𝑏 + 30 (𝑏 + 𝑐) + 2 ]
𝑎 + 𝑎𝑏 + 𝑏𝑐 √∏ (𝑎2 + 𝑎𝑏 + 𝑏𝑐)
3

3 3
33𝑎𝑏𝑐 99𝑎𝑏𝑐
≥ [10√𝑎𝑏𝑐 + 30√3 ∏ (𝑏 + 𝑐) + ] = [10√𝑎𝑏𝑐 + 30√3 (𝑎 + 𝑏) (𝑏 + 𝑐) (𝑐 + 𝑎) +
3 3
∑(𝑎2 +𝑎𝑏+𝑏𝑐) 2
] ,
3 (𝑎 + 𝑏 + 𝑐)
nên chỉ cần chứng minh được
99𝑎𝑏𝑐
≥ 71√𝑎𝑏𝑐,
3
30√3 (𝑎 + 𝑏) (𝑏 + 𝑐) (𝑐 + 𝑎) + 2
(𝑎 + 𝑏 + 𝑐)
3 (𝑎 + 𝑏) (𝑏 + 𝑐) (𝑐 + 𝑎) 9𝑎𝑏𝑐
60√ ≥ 71√𝑎𝑏𝑐.
3
+ 11 ⋅
8 (𝑎 + 𝑏 + 𝑐)
2

Thật vậy, tiếp tục sử dụng bất đẳng thức AM – GM,

60 11 20 11
3 (𝑎 + 𝑏) (𝑏 + 𝑐) (𝑐 + 𝑎)
71 9𝑎𝑏𝑐 (𝑎 + 𝑏) (𝑏 + 𝑐) (𝑐 + 𝑎) (9𝑎𝑏𝑐)
𝑉𝛵 ≥ 71 √[√ ] [ ] = 71 71√[ ] ⋅
8 (𝑎 + 𝑏 + 𝑐)
2 8 (𝑎 + 𝑏 + 𝑐)
22

20
20 11 11
(𝑎 + 𝑏 + 𝑐) (𝑎𝑏 + 𝑏𝑐 + 𝑐𝑎) (9𝑎𝑏𝑐) 71 (𝑎 + 𝑏 + 𝑐) √3𝑎𝑏𝑐 (𝑎 + 𝑏 + 𝑐) (9𝑎𝑏𝑐)
≥ 71 √[
71
] ⋅ ≥ 71 √[ ] ⋅
9 (𝑎 + 𝑏 + 𝑐)
22 9 (𝑎 + 𝑏 + 𝑐)
22

8
21 𝑎+𝑏+𝑐 21 3 8
) ≥ 71 √(𝑎𝑏𝑐) (√𝑎𝑏𝑐) = 71√𝑎𝑏𝑐.
71
= 71 √(𝑎𝑏𝑐) (
71 3

3
Đẳng thức xảy ra khi 𝑎 = 𝑏 = 𝑐 = 1. 

Bài 2.3. Cho các số thực dương 𝑎, 𝑏, 𝑐, chứng minh rằng


𝑎+𝑏+𝑐 𝑎 𝑎+𝑏+𝑐
≥∑ 2 ≥ .
𝑎𝑏 + 𝑏𝑐 + 𝑐𝑎 𝑎 + 2𝑏𝑐 𝑎2 + 𝑏2 + 𝑐2

Lời giải. Dấu ≥ thứ hai được suy ra trực tiếp từ bất đẳng thức 2𝑥𝑦 ≤ 𝑥2 + 𝑦2 ,
𝑎 𝑎 𝑎+𝑏+𝑐
∑ ≥∑ 2 = 2 .
𝑎2 + 2𝑏𝑐 2
𝑎 +𝑏 +𝑐 2 𝑎 + 𝑏 2 + 𝑐2
Với dấu ≥ thứ nhất, giả sử 𝑎 ≥ 𝑏 ≥ 𝑐 ≥ 0, sau đó nhân hai vế của bất đẳng thức với 𝑎𝑏 + 𝑏𝑐 + 𝑐𝑎 và viết lại dưới dạng tương
đương là
𝑎 (𝑎𝑏 + 𝑏𝑐 + 𝑐𝑎)
∑ (𝑎 − ) ≥ 0,
𝑎2 + 2𝑏𝑐
𝑎 (𝑎 − 𝑏) (𝑎 − 𝑐)
∑ ≥ 0,
𝑎2 + 2𝑏𝑐
𝑎 (𝑎 − 𝑏) (𝑎 − 𝑐) 𝑏 (𝑏 − 𝑐) (𝑏 − 𝑎) 𝑐 (𝑐 − 𝑎) (𝑐 − 𝑏)
+ + ≥ 0,
𝑎2 + 2𝑏𝑐 𝑏2 + 2𝑐𝑎 𝑐2 + 2𝑎𝑏
𝑎 (𝑎 − 𝑐) 𝑏 (𝑏 − 𝑐) 𝑐 (𝑐 − 𝑎) (𝑐 − 𝑏)
(𝑎 − 𝑏) ( 2 − )+ ≥ 0,
𝑎 + 2𝑏𝑐 𝑏2 + 2𝑐𝑎 𝑐2 + 2𝑎𝑏
2
𝑐 (𝑎 − 𝑏) [3𝑎𝑏 + 2𝑎 (𝑎 − 𝑐) + 2𝑏 (𝑏 − 𝑐)] 𝑐 (𝑐 − 𝑎) (𝑐 − 𝑏)
+ ≥ 0.
(𝑎2 + 2𝑏𝑐) (𝑏2 + 2𝑐𝑎) 𝑐2 + 2𝑎𝑏
Bất đẳng thức cuối này hiển nhiên đúng với 𝑎 ≥ 𝑏 ≥ 𝑐 ≥ 0. 

Đỗ Xuân Trọng 11
CHƯƠNG 2. NHỮNG BÀI TOÁN LÍ THÚ XOAY QUANH MỘT ĐẠI LƯỢNG HOÁN VỊ

Bài 2.4. Cho các số thực dương 𝑎, 𝑏, 𝑐, chứng minh rằng


𝑎+𝑏+𝑐 𝑎 𝑎+𝑏+𝑐
≥∑ 2 2
≥ 2 .
𝑎𝑏 + 𝑏𝑐 + 𝑐𝑎 𝑎 + 𝑎𝑏 + 𝑏 𝑎 + 𝑏2 + 𝑐2

Lời giải. Dấu ≥ thứ hai được suy ra từ bất đẳng thức Cauchy – Schwarz,
2 2
𝑎 𝑎2 (𝑎 + 𝑏 + 𝑐) (𝑎 + 𝑏 + 𝑐) 𝑎+𝑏+𝑐
∑ =∑ 3 ≥ = = .
𝑎2 + 𝑎𝑏 + 𝑏 2 𝑎 + 𝑎2 𝑏 + 𝑎𝑏2 ∑ (𝑎3 + 𝑎2 𝑏 + 𝑎𝑏2 ) (𝑎 + 𝑏 + 𝑐) (𝑎2 + 𝑏2 + 𝑐2 ) 𝑎2 + 𝑏2 + 𝑐2

Với dấu ≥ thứ nhất, nhân hai vế của bất đẳng thức với 𝑎2 + 𝑏2 + 𝑐2 + 𝑎𝑏 + 𝑏𝑐 + 𝑐𝑎, ta có

(𝑎 + 𝑏 + 𝑐) (𝑎2 + 𝑏2 + 𝑐2 + 𝑎𝑏 + 𝑏𝑐 + 𝑐𝑎) 𝑎 (𝑎2 + 𝑏2 + 𝑐2 + 𝑎𝑏 + 𝑏𝑐 + 𝑐𝑎)


≥∑ ,
𝑎𝑏 + 𝑏𝑐 + 𝑐𝑎 𝑎2 + 𝑎𝑏 + 𝑏2
(𝑎 + 𝑏 + 𝑐) (𝑎2 + 𝑏2 + 𝑐2 ) 𝑎𝑐 (𝑎 + 𝑏 + 𝑐)
𝑎+𝑏+𝑐+ ≥ ∑ [𝑎 + 2 ],
𝑎𝑏 + 𝑏𝑐 + 𝑐𝑎 𝑎 + 𝑎𝑏 + 𝑏2
(𝑎 + 𝑏 + 𝑐) (𝑎2 + 𝑏2 + 𝑐2 ) 𝑎𝑐
𝑎+𝑏+𝑐+ ≥ ∑ 𝑎 + (∑ 𝑎) (∑ 2 ),
𝑎𝑏 + 𝑏𝑐 + 𝑐𝑎 𝑎 + 𝑎𝑏 + 𝑏2
𝑎2 + 𝑏 2 + 𝑐 2 𝑎𝑐
≥∑ 2 .
𝑎𝑏 + 𝑏𝑐 + 𝑐𝑎 𝑎 + 𝑎𝑏 + 𝑏2
Đây là bài 6 ở chuyên đề 2 của sách (xem trang 234). 

Bài 2.5. Cho các số thực dương 𝑎, 𝑏, 𝑐, chứng minh rằng


𝑎+𝑏+𝑐 𝑎 𝑎+𝑏+𝑐
≥∑ 2 ≥ 2 .
𝑎𝑏 + 𝑏𝑐 + 𝑐𝑎 𝑎 + 𝑎𝑏 + 𝑏𝑐 𝑎 + 𝑏2 + 𝑐2

Lời giải. Với dấu ≥ thứ hai, sử dụng bất đẳng thức AM – GM, dễ thấy

𝑎𝑏2 + 𝑏𝑐2 + 𝑐𝑎2 ≥ 3𝑎𝑏𝑐,

nên theo bất đẳng thức Cauchy – Schwarz,


2 2
𝑎 𝑎2 (𝑎 + 𝑏 + 𝑐) (𝑎 + 𝑏 + 𝑐)
∑ 2
=∑ 3 ≥ = ≥
𝑎 + 𝑎𝑏 + 𝑏𝑐 𝑎 + 𝑎2 𝑏 + 𝑎𝑏𝑐 ∑ (𝑎3 + 𝑎2 𝑏 + 𝑎𝑏𝑐) ∑ 𝑎3 + ∑ 𝑎2 𝑏 + 3𝑎𝑏𝑐
2 2
(𝑎 + 𝑏 + 𝑐) (𝑎 + 𝑏 + 𝑐) 𝑎+𝑏+𝑐
≥ = = .
∑ 𝑎3 + ∑ 𝑎2 𝑏 + ∑ 𝑎𝑏2 (𝑎 + 𝑏 + 𝑐) (𝑎2 + 𝑏2 + 𝑐2 ) 𝑎2 + 𝑏2 + 𝑐2
Với dấu ≥ thứ nhất, nhân hai vế của bất đẳng thức với 𝑎 + 𝑏 + 𝑐, ta có
2
(𝑎 + 𝑏 + 𝑐) 𝑎 (𝑎 + 𝑏 + 𝑐)
≥∑ 2 ,
𝑎𝑏 + 𝑏𝑐 + 𝑐𝑎 𝑎 + 𝑎𝑏 + 𝑏𝑐
2
(𝑎 + 𝑏 + 𝑐) 𝑎 (𝑎 + 𝑏 + 𝑐)
≥ ∑( 2 − 1) ,
𝑎𝑏 + 𝑏𝑐 + 𝑐𝑎 𝑎 + 𝑎𝑏 + 𝑏𝑐
𝑎2 + 𝑏 2 + 𝑐 2 𝑎𝑐 − 𝑏𝑐
−1≥∑ 2 ,
𝑎𝑏 + 𝑏𝑐 + 𝑐𝑎 𝑎 + 𝑎𝑏 + 𝑏𝑐
𝑎2 + 𝑏 2 + 𝑐 2 𝑏𝑐 𝑎𝑐
+∑ 2 ≥1+∑ 2 .
𝑎𝑏 + 𝑏𝑐 + 𝑐𝑎 𝑎 + 𝑎𝑏 + 𝑏𝑐 𝑎 + 𝑎𝑏 + 𝑏𝑐

Đỗ Xuân Trọng 12
CHƯƠNG 2. NHỮNG BÀI TOÁN LÍ THÚ XOAY QUANH MỘT ĐẠI LƯỢNG HOÁN VỊ

Theo bất đẳng thức Cauchy – Schwarz,


2 2
𝑏𝑐 𝑏2 𝑐2 (𝑏𝑐 + 𝑐𝑎 + 𝑎𝑏) (𝑎𝑏 + 𝑏𝑐 + 𝑐𝑎)
∑ =∑ 2 ≥ = = 1,
𝑎2 + 𝑎𝑏 + 𝑏𝑐 𝑎 𝑏𝑐 + 𝑎𝑏2 𝑐 + 𝑏2 𝑐2 ∑ (𝑎2 𝑏𝑐 + 𝑎𝑏2 𝑐 + 𝑏2 𝑐2 ) ∑ 𝑎2 𝑏2 + 2𝑎𝑏𝑐 ∑ 𝑎
nên ta chỉ cần chứng minh
𝑎2 + 𝑏 2 + 𝑐 2 𝑎𝑐
≥∑ 2 . (2.2)
𝑎𝑏 + 𝑏𝑐 + 𝑐𝑎 𝑎 + 𝑎𝑏 + 𝑏𝑐
Tiếp tục sử dụng bất đẳng thức Cauchy – Schwarz, ta có

𝑎𝑐 𝑎𝑐 (𝑐2 + 𝑎𝑏 + 𝑏𝑐) 𝑎𝑐 (𝑐2 + 𝑎𝑏 + 𝑏𝑐) 𝑎3 𝑏 + 𝑏3 𝑐 + 𝑐3 𝑎 + 2𝑎𝑏𝑐 (𝑎 + 𝑏 + 𝑐)


∑ = ∑ ≤ ∑ = ,
𝑎2 + 𝑎𝑏 + 𝑏𝑐 (𝑎2 + 𝑎𝑏 + 𝑏𝑐) (𝑐2 + 𝑎𝑏 + 𝑏𝑐) (𝑎𝑐 + 𝑎𝑏 + 𝑏𝑐)
2
(𝑎𝑏 + 𝑏𝑐 + 𝑐𝑎)
2

nên ta lại quy (2.2) về chứng minh

𝑎2 + 𝑏2 + 𝑐2 𝑎3 𝑏 + 𝑏3 𝑐 + 𝑐3 𝑎 + 2𝑎𝑏𝑐 (𝑎 + 𝑏 + 𝑐)
≥ 2
,
𝑎𝑏 + 𝑏𝑐 + 𝑐𝑎 (𝑎𝑏 + 𝑏𝑐 + 𝑐𝑎)

(𝑎𝑏 + 𝑏𝑐 + 𝑐𝑎) (𝑎2 + 𝑏2 + 𝑐2 ) ≥ 𝑎3 𝑏 + 𝑏3 𝑐 + 𝑐3 𝑎 + 2𝑎𝑏𝑐 (𝑎 + 𝑏 + 𝑐) ,


𝑎𝑏3 + 𝑏𝑐3 + 𝑐𝑎3 ≥ 𝑎𝑏𝑐 (𝑎 + 𝑏 + 𝑐) ,
𝑏2 𝑐2 𝑎2
+ + ≥ 𝑎 + 𝑏 + 𝑐.
𝑐 𝑎 𝑏
Bất đẳng thức cuối này lại tiếp tục đúng theo bất đẳng thức Cauchy – Schwarz,
2
𝑏2 𝑐2 𝑎2 (𝑏 + 𝑐 + 𝑎)
+ + ≥ = 𝑎 + 𝑏 + 𝑐.
𝑐 𝑎 𝑏 𝑐+𝑎+𝑏
Bài toán kết thúc. 

Bài 2.6. Cho các số thực dương 𝑎, 𝑏, 𝑐, chứng minh rằng


𝑎+𝑏+𝑐 𝑎 𝑎+𝑏+𝑐
≥∑ 2 2
≥ 2 .
𝑎𝑏 + 𝑏𝑐 + 𝑐𝑎 𝑎 + 𝑏𝑐 + 𝑏 𝑎 + 𝑏2 + 𝑐2

Lời giải. Với dấu ≥ thứ hai, theo bất đẳng thức AM – GM,

𝑎2 𝑏 + 𝑏2 𝑐 + 𝑐2 𝑎 ≥ 3𝑎𝑏𝑐,

nên theo bất đẳng thức Cauchy – Schwarz,


2 2
𝑎 𝑎2 (𝑎 + 𝑏 + 𝑐) (𝑎 + 𝑏 + 𝑐)
∑ 2 2
=∑ 3 ≥ =
𝑎 + 𝑏𝑐 + 𝑏 𝑎 + 𝑎𝑏𝑐 + 𝑎𝑏2 ∑ (𝑎3 + 𝑎𝑏𝑐 + 𝑎𝑏2 ) ∑ 𝑎3 + ∑ 𝑎𝑏2 + 3𝑎𝑏𝑐
2 2
(𝑎 + 𝑏 + 𝑐) (𝑎 + 𝑏 + 𝑐) 𝑎+𝑏+𝑐
≥ = = 2 .
∑ 𝑎 + ∑ 𝑎𝑏 + ∑ 𝑎 𝑏 (𝑎 + 𝑏 + 𝑐) (𝑎 + 𝑏 + 𝑐 ) 𝑎 + 𝑏2 + 𝑐2
3 2 2 2 2 2

Với dấu ≥ thứ nhất, quy đồng mẫu số chung

(𝑎𝑏 + 𝑏𝑐 + 𝑐𝑎) (𝑎2 + 𝑏𝑐 + 𝑏2 ) (𝑏2 + 𝑐𝑎 + 𝑐2 ) (𝑐2 + 𝑎𝑏 + 𝑎2 )

cho ta bất đẳng thức tương đương

∑ 𝑎𝑏6 + ∑ 𝑎2 𝑏5 + 𝑎2 𝑏2 𝑐2 ∑ 𝑎 ≥ 2𝑎𝑏𝑐 ∑ 𝑎2 𝑏2 + 𝑎𝑏𝑐 ∑ 𝑎3 𝑏,

Đỗ Xuân Trọng 13
CHƯƠNG 2. NHỮNG BÀI TOÁN LÍ THÚ XOAY QUANH MỘT ĐẠI LƯỢNG HOÁN VỊ

∑ 𝑎𝑏6 + ∑ 𝑎2 𝑏5 + 𝑎𝑏𝑐 (∑ 𝑎4 + 𝑎𝑏𝑐 ∑ 𝑎 − 2 ∑ 𝑎2 𝑏2 ) ≥ 𝑎𝑏𝑐 (∑ 𝑎4 + ∑ 𝑎3 𝑏) . (2.3)

Khai triển bất đẳng thức Schur bậc 4


∑ 𝑎2 (𝑎 − 𝑏) (𝑎 − 𝑐) ≥ 0

cho ta
∑ 𝑎4 + 𝑎𝑏𝑐 ∑ 𝑎 ≥ ∑ 𝑎𝑏 (𝑎2 + 𝑏2 ) .

Mặt khác, vì 𝑥2 + 𝑦2 ≥ 2𝑥𝑦 nên suy ra

∑ 𝑎4 + 𝑎𝑏𝑐 ∑ 𝑎 ≥ ∑ 𝑎𝑏 (𝑎2 + 𝑏2 ) ≥ ∑ 𝑎𝑏 (2𝑎𝑏) = 2 ∑ 𝑎2 𝑏2 .

Do đó, (2.3) sẽ được chứng minh, nếu ta chứng minh được

∑ 𝑎𝑏6 + ∑ 𝑎2 𝑏5 ≥ 𝑎𝑏𝑐 (∑ 𝑎4 + ∑ 𝑎3 𝑏) .

Chia hai vế cho 𝑎𝑏𝑐, ta có


𝑏5 𝑎𝑏4
∑ +∑ ≥ ∑ 𝑎4 + ∑ 𝑎3 𝑏. (2.4)
𝑐 𝑐
Theo bất đẳng thức AM – GM,
𝑏5
∑( + 𝑏3 𝑐) ≥ ∑ 2𝑏4 = 2 ∑ 𝑎4 ,
𝑐
𝑏5
∑ ≥ 2 ∑ 𝑎4 − ∑ 𝑎3 𝑏.
𝑐
Do đó, ta quy (2.4) về
𝑎𝑏4
∑ 𝑎4 + ∑ ≥ 2 ∑ 𝑎3 𝑏.
𝑐
Theo bất đẳng thức AM – GM,

4 4
4𝑎𝑏4 4𝑐𝑎4 7 𝑎𝑏4 𝑐𝑎4
≥ 16 √(𝑎4 ) ⋅ 𝑏4 (
16
7𝑎4 + 𝑏4 + + ) ( ) = 16𝑎3 𝑏. (2.5)
𝑐 𝑏 𝑐 𝑏

Tương tự,
4𝑏𝑐4 4𝑎𝑏4
7𝑏4 + 𝑐4 + + ≥ 16𝑏3 𝑐, (2.6)
𝑎 𝑐
4𝑐𝑎4 4𝑏𝑐4
7𝑐4 + 𝑎4 + + ≥ 16𝑐3 𝑎. (2.7)
𝑏 𝑎
Cộng các đánh giá (2.5), (2.6), (2.7) theo vế, rồi chia cả hai vế cho 8, ta có điều phải chứng minh. 

Đỗ Xuân Trọng 14

You might also like